Calculating the gradient of a logarithmic scale

Click For Summary
To calculate the gradient of a logarithmic scale in MATLAB, one must use the change in the logarithmic values rather than the original numbers. A log-log plot transforms the data to (log(NX), log(plotERRLW)), which allows for a linear relationship if the data follows a power law. The gradient can be determined by taking the log of both the x and y values, resulting in a straight line equation of the form log(y) = a log(x) + b. The logarithm base used in MATLAB's loglog function is base 10 by default. For accurate results, ensure that the logarithmic transformation aligns with the intended mathematical model.
blizzard12345
Messages
11
Reaction score
0

Homework Statement



hi i have the folowing data i would like to plot in matlab

plotERRLW =

0.0466 0.0111 0.0074 0.0046
NX =

50 500 1000 2000
i am using a logarithmic graph to gain a straight line, if i wished to find the gradient of the line would i use the change in the original numbers or change in the numbers after i have taken a log of them?

(also if i have to take a log of them what base does loglog naturally plot)

i have tried a number of options already but can't seem to get the correct answer (i know it should be close to 2) thanks in advance



Homework Equations



i believe the data is of the form plotERRLW = NX^(alpha)



The Attempt at a Solution



so far i have been taking the log base 10 of the numbers in question and simply dividing the log of the error with the log of NX

thanks in advance kyle
 
Physics news on Phys.org
A log plot of (x, y) is a plot of (x, log(y)). If it is a straight line, they you are saying that log(y)= ax+b so that y= e^{ax+b}= e^be^{ax}=Ce^{ax}, a general exponential function.

A "log-log" plot of (x, y) is a plot of (log(x), log(y)). If a log-log plot gives a straight line through the origin, then log(y)= a log(x) so that y= x^a.
 
Question: A clock's minute hand has length 4 and its hour hand has length 3. What is the distance between the tips at the moment when it is increasing most rapidly?(Putnam Exam Question) Answer: Making assumption that both the hands moves at constant angular velocities, the answer is ## \sqrt{7} .## But don't you think this assumption is somewhat doubtful and wrong?

Similar threads

  • · Replies 2 ·
Replies
2
Views
14K
  • · Replies 1 ·
Replies
1
Views
1K
  • · Replies 9 ·
Replies
9
Views
5K
  • · Replies 3 ·
Replies
3
Views
5K
  • · Replies 16 ·
Replies
16
Views
2K
  • · Replies 7 ·
Replies
7
Views
2K
  • · Replies 1 ·
Replies
1
Views
1K
  • · Replies 3 ·
Replies
3
Views
2K
  • · Replies 4 ·
Replies
4
Views
3K
Replies
3
Views
2K